Topological invariance of dimension

The name of the pictureThe name of the pictureThe name of the pictureClash Royale CLAN TAG#URR8PPP











up vote
3
down vote

favorite












I am starting to study smooth manifolds with the book of Lee. At the beginning, he states this theorem, which is then proven later on with advanced techniques:



Theorem 1.2 (Topological Invariance of Dimension). A nonempty $n$-dimensional topological manifold cannot be homeomorphic to an $m$-dimensional manifold unless $m = n$.



I imagine that this is because there is no short simple proof. So I would like to know what is wrong with the following argument:



Let $M$ be a space, and $x in M$ such that there are open neighbourhoods $U, V$ of $x$ which are homeomorphic to open subsets of $mathbbR^n$ and $mathbbR^m$, respectively (note furthermore that here the hypothesis is much weaker than being a topological manifold, so there must really be something wrong).
Then $U cap V$ is an open subset of both $U$ and $V$, which implies that under the respective homeomorphisms, $U cap V$ is homeomorphic to both an open subset of $mathbbR^n$ and an open subset of $mathbbR^m$. Also, $x in U cap V$, so $U cap V neq emptyset$. This however is possible only if $m = n$.



Thank you in advance.







share|cite|improve this question
























    up vote
    3
    down vote

    favorite












    I am starting to study smooth manifolds with the book of Lee. At the beginning, he states this theorem, which is then proven later on with advanced techniques:



    Theorem 1.2 (Topological Invariance of Dimension). A nonempty $n$-dimensional topological manifold cannot be homeomorphic to an $m$-dimensional manifold unless $m = n$.



    I imagine that this is because there is no short simple proof. So I would like to know what is wrong with the following argument:



    Let $M$ be a space, and $x in M$ such that there are open neighbourhoods $U, V$ of $x$ which are homeomorphic to open subsets of $mathbbR^n$ and $mathbbR^m$, respectively (note furthermore that here the hypothesis is much weaker than being a topological manifold, so there must really be something wrong).
    Then $U cap V$ is an open subset of both $U$ and $V$, which implies that under the respective homeomorphisms, $U cap V$ is homeomorphic to both an open subset of $mathbbR^n$ and an open subset of $mathbbR^m$. Also, $x in U cap V$, so $U cap V neq emptyset$. This however is possible only if $m = n$.



    Thank you in advance.







    share|cite|improve this question






















      up vote
      3
      down vote

      favorite









      up vote
      3
      down vote

      favorite











      I am starting to study smooth manifolds with the book of Lee. At the beginning, he states this theorem, which is then proven later on with advanced techniques:



      Theorem 1.2 (Topological Invariance of Dimension). A nonempty $n$-dimensional topological manifold cannot be homeomorphic to an $m$-dimensional manifold unless $m = n$.



      I imagine that this is because there is no short simple proof. So I would like to know what is wrong with the following argument:



      Let $M$ be a space, and $x in M$ such that there are open neighbourhoods $U, V$ of $x$ which are homeomorphic to open subsets of $mathbbR^n$ and $mathbbR^m$, respectively (note furthermore that here the hypothesis is much weaker than being a topological manifold, so there must really be something wrong).
      Then $U cap V$ is an open subset of both $U$ and $V$, which implies that under the respective homeomorphisms, $U cap V$ is homeomorphic to both an open subset of $mathbbR^n$ and an open subset of $mathbbR^m$. Also, $x in U cap V$, so $U cap V neq emptyset$. This however is possible only if $m = n$.



      Thank you in advance.







      share|cite|improve this question












      I am starting to study smooth manifolds with the book of Lee. At the beginning, he states this theorem, which is then proven later on with advanced techniques:



      Theorem 1.2 (Topological Invariance of Dimension). A nonempty $n$-dimensional topological manifold cannot be homeomorphic to an $m$-dimensional manifold unless $m = n$.



      I imagine that this is because there is no short simple proof. So I would like to know what is wrong with the following argument:



      Let $M$ be a space, and $x in M$ such that there are open neighbourhoods $U, V$ of $x$ which are homeomorphic to open subsets of $mathbbR^n$ and $mathbbR^m$, respectively (note furthermore that here the hypothesis is much weaker than being a topological manifold, so there must really be something wrong).
      Then $U cap V$ is an open subset of both $U$ and $V$, which implies that under the respective homeomorphisms, $U cap V$ is homeomorphic to both an open subset of $mathbbR^n$ and an open subset of $mathbbR^m$. Also, $x in U cap V$, so $U cap V neq emptyset$. This however is possible only if $m = n$.



      Thank you in advance.









      share|cite|improve this question











      share|cite|improve this question




      share|cite|improve this question










      asked Aug 25 at 16:40









      user404944

      58418




      58418




















          1 Answer
          1






          active

          oldest

          votes

















          up vote
          6
          down vote













          Your proof seems to use that $mathbbR^m$ is homeomorphic to $mathbbR^n$ if, and only, if $n=m$, but this statement is highly non-trivial (seems intuitive though) and equivalent to the one for topological manifolds (it is actually the core of the topological invariance of the dimension).



          With easy tools (connexity argument), you can prove that $mathbbR^n$ is homeomorphic to $mathbbR$ if, and only, if $n=1$.



          As a side note, can you establish with a one-line and really easy proof that $mathbbR^m$ is diffeomorphic to $mathbbR^n$ if, and only, if $n=m$?






          share|cite|improve this answer


















          • 3




            (+1) I would add that this part is indeed the only non-trivial part of the proof.
            – Arnaud Mortier
            Aug 25 at 16:53











          Your Answer




          StackExchange.ifUsing("editor", function ()
          return StackExchange.using("mathjaxEditing", function ()
          StackExchange.MarkdownEditor.creationCallbacks.add(function (editor, postfix)
          StackExchange.mathjaxEditing.prepareWmdForMathJax(editor, postfix, [["$", "$"], ["\\(","\\)"]]);
          );
          );
          , "mathjax-editing");

          StackExchange.ready(function()
          var channelOptions =
          tags: "".split(" "),
          id: "69"
          ;
          initTagRenderer("".split(" "), "".split(" "), channelOptions);

          StackExchange.using("externalEditor", function()
          // Have to fire editor after snippets, if snippets enabled
          if (StackExchange.settings.snippets.snippetsEnabled)
          StackExchange.using("snippets", function()
          createEditor();
          );

          else
          createEditor();

          );

          function createEditor()
          StackExchange.prepareEditor(
          heartbeatType: 'answer',
          convertImagesToLinks: true,
          noModals: false,
          showLowRepImageUploadWarning: true,
          reputationToPostImages: 10,
          bindNavPrevention: true,
          postfix: "",
          noCode: true, onDemand: true,
          discardSelector: ".discard-answer"
          ,immediatelyShowMarkdownHelp:true
          );



          );













           

          draft saved


          draft discarded


















          StackExchange.ready(
          function ()
          StackExchange.openid.initPostLogin('.new-post-login', 'https%3a%2f%2fmath.stackexchange.com%2fquestions%2f2894287%2ftopological-invariance-of-dimension%23new-answer', 'question_page');

          );

          Post as a guest






























          1 Answer
          1






          active

          oldest

          votes








          1 Answer
          1






          active

          oldest

          votes









          active

          oldest

          votes






          active

          oldest

          votes








          up vote
          6
          down vote













          Your proof seems to use that $mathbbR^m$ is homeomorphic to $mathbbR^n$ if, and only, if $n=m$, but this statement is highly non-trivial (seems intuitive though) and equivalent to the one for topological manifolds (it is actually the core of the topological invariance of the dimension).



          With easy tools (connexity argument), you can prove that $mathbbR^n$ is homeomorphic to $mathbbR$ if, and only, if $n=1$.



          As a side note, can you establish with a one-line and really easy proof that $mathbbR^m$ is diffeomorphic to $mathbbR^n$ if, and only, if $n=m$?






          share|cite|improve this answer


















          • 3




            (+1) I would add that this part is indeed the only non-trivial part of the proof.
            – Arnaud Mortier
            Aug 25 at 16:53















          up vote
          6
          down vote













          Your proof seems to use that $mathbbR^m$ is homeomorphic to $mathbbR^n$ if, and only, if $n=m$, but this statement is highly non-trivial (seems intuitive though) and equivalent to the one for topological manifolds (it is actually the core of the topological invariance of the dimension).



          With easy tools (connexity argument), you can prove that $mathbbR^n$ is homeomorphic to $mathbbR$ if, and only, if $n=1$.



          As a side note, can you establish with a one-line and really easy proof that $mathbbR^m$ is diffeomorphic to $mathbbR^n$ if, and only, if $n=m$?






          share|cite|improve this answer


















          • 3




            (+1) I would add that this part is indeed the only non-trivial part of the proof.
            – Arnaud Mortier
            Aug 25 at 16:53













          up vote
          6
          down vote










          up vote
          6
          down vote









          Your proof seems to use that $mathbbR^m$ is homeomorphic to $mathbbR^n$ if, and only, if $n=m$, but this statement is highly non-trivial (seems intuitive though) and equivalent to the one for topological manifolds (it is actually the core of the topological invariance of the dimension).



          With easy tools (connexity argument), you can prove that $mathbbR^n$ is homeomorphic to $mathbbR$ if, and only, if $n=1$.



          As a side note, can you establish with a one-line and really easy proof that $mathbbR^m$ is diffeomorphic to $mathbbR^n$ if, and only, if $n=m$?






          share|cite|improve this answer














          Your proof seems to use that $mathbbR^m$ is homeomorphic to $mathbbR^n$ if, and only, if $n=m$, but this statement is highly non-trivial (seems intuitive though) and equivalent to the one for topological manifolds (it is actually the core of the topological invariance of the dimension).



          With easy tools (connexity argument), you can prove that $mathbbR^n$ is homeomorphic to $mathbbR$ if, and only, if $n=1$.



          As a side note, can you establish with a one-line and really easy proof that $mathbbR^m$ is diffeomorphic to $mathbbR^n$ if, and only, if $n=m$?







          share|cite|improve this answer














          share|cite|improve this answer



          share|cite|improve this answer








          edited Aug 25 at 16:55

























          answered Aug 25 at 16:46









          C. Falcon

          14.7k41748




          14.7k41748







          • 3




            (+1) I would add that this part is indeed the only non-trivial part of the proof.
            – Arnaud Mortier
            Aug 25 at 16:53













          • 3




            (+1) I would add that this part is indeed the only non-trivial part of the proof.
            – Arnaud Mortier
            Aug 25 at 16:53








          3




          3




          (+1) I would add that this part is indeed the only non-trivial part of the proof.
          – Arnaud Mortier
          Aug 25 at 16:53





          (+1) I would add that this part is indeed the only non-trivial part of the proof.
          – Arnaud Mortier
          Aug 25 at 16:53


















           

          draft saved


          draft discarded















































           


          draft saved


          draft discarded














          StackExchange.ready(
          function ()
          StackExchange.openid.initPostLogin('.new-post-login', 'https%3a%2f%2fmath.stackexchange.com%2fquestions%2f2894287%2ftopological-invariance-of-dimension%23new-answer', 'question_page');

          );

          Post as a guest













































































          Comments

          Popular posts from this blog

          Long meetings (6-7 hours a day): Being “babysat” by supervisor

          Is the Concept of Multiple Fantasy Races Scientifically Flawed? [closed]

          Confectionery